Hermitian Operators and Inner Products

Click For Summary
SUMMARY

The discussion centers on proving that the operator \(\hat{L_z} = -i \hbar \left(x \frac{\delta}{\delta y} - y \frac{\delta}{\delta x}\right)\) is a self-adjoint (Hermitian) operator in the vector space of square-integrable functions \(\psi(x,y,z)\). The inner product is defined as \(\langle \phi | \psi \rangle = \int_{-\infty}^{\infty} \int_{-\infty}^{\infty} \int_{-\infty}^{\infty} \phi^* \psi \, dx \, dy \, dz\). To demonstrate that \(\hat{L_z}^\dagger = \hat{L_z}\), one must show that \(\langle \hat{L_z} \phi | \psi \rangle = \langle \phi | \hat{L_z} \psi \rangle\) using integration by parts, confirming the operator's Hermitian nature.

PREREQUISITES
  • Understanding of Hermitian operators in quantum mechanics
  • Familiarity with inner products in functional spaces
  • Knowledge of partial differentiation and integration techniques
  • Basic concepts of linear algebra and operator theory
NEXT STEPS
  • Study the properties of Hermitian operators in quantum mechanics
  • Learn about integration by parts in the context of functional analysis
  • Explore the implications of self-adjoint operators in quantum mechanics
  • Investigate the role of square-integrable functions in quantum theory
USEFUL FOR

Students and professionals in physics, particularly those focusing on quantum mechanics, linear algebra, and functional analysis, will benefit from this discussion. It is especially relevant for those studying operator theory and its applications in quantum systems.

csnsc14320
Messages
57
Reaction score
1

Homework Statement



Consider the vector space of square-integrable functions \psi(x,y,z) of (real space) position {x,y,z} where \psi vanishes at infinity in all directions. Define the inner product for this space to be

<\phi|\psi> = \int_{-\infty}^{\infty} \int_{-\infty}^{\infty} \int_{-\infty}^{\infty} \phi^* \psi dx dy dz

Show that the operator

\hat{L_z} = -i \hbar \left(x \frac{\delta}{\delta y} - y \frac{\delta}{\delta x}\right)

is a self-adjoint (Hermitian) operator on this space.

Homework Equations


The Attempt at a Solution



So I know that for an operator to be Hermitian that \hat{H} = \hat{H}^\dagger

so just starting from what I know...

<\hat{L} \phi|\psi> = <\phi|\hat{L}^\dagger|\psi>

<\hat{L} \phi|\psi> = \int_{-\infty}^{\infty} \int_{-\infty}^{\infty} \int_{-\infty}^{\infty} \left(-i \hbar \left(x \frac{\delta}{\delta y} - y \frac{\delta}{\delta x}\right) \phi \right)^* \psi dx dy dz = \int_{-\infty}^{\infty} \int_{-\infty}^{\infty} \int_{-\infty}^{\infty} i \hbar \left(x \frac{\delta}{\delta y} - y \frac{\delta}{\delta x}\right) \phi^* \psi dx dy dz

and

<\phi|\hat{L}^\dagger|\psi> = \int_{-\infty}^{\infty} \int_{-\infty}^{\infty} \int_{-\infty}^{\infty} \phi^* \left(i \hbar \left(x \frac{\delta}{\delta y} - y \frac{\delta}{\delta x}\right)\psi \right) dx dy dz = \int_{-\infty}^{\infty} \int_{-\infty}^{\infty} \int_{-\infty}^{\infty} i \hbar \left(x \frac{\delta}{\delta y} - y \frac{\delta}{\delta x}\right) \phi^* \psi dx dy dz

But I guess these are my two main questions:

For a non-matrix operator

\hat{L_z} = -i \hbar \left(x \frac{\delta}{\delta y} - y \frac{\delta}{\delta x}\right)

does

\hat{L_z}^\dagger = i \hbar \left(x \frac{\delta}{\delta y} - y \frac{\delta}{\delta x}\right)

??

and, can I move around my partials in the second equation to match that of my first or do I have to carry them out?

Thanks, and I'm sorry if what I said is complete junk seeing as how I'm not THAT strong in linear algebra... yet.
 
Physics news on Phys.org
csnsc14320 said:
But I guess these are my two main questions:

For a non-matrix operator

\hat{L_z} = -i \hbar \left(x \frac{\delta}{\delta y} - y \frac{\delta}{\delta x}\right)

does

\hat{L_z}^\dagger = i \hbar \left(x \frac{\delta}{\delta y} - y \frac{\delta}{\delta x}\right)
Actually, L_z^\dagger = L_z -- that is what you are trying to show (it's precisely what being Hermitian means).

So the idea is that you start from
\langle L_z \phi \mid \psi \rangle
where L works on the first function, and use partial integration to show that it is equal to
\langle \phi \mid L_z \psi \rangle
where L works on the second function.

(Then, of course, since you know that it is also equal to \langle \phi \mid L_z^\dagger \psi \rangle you can conclude that L is equal to its Hermitian conjugate and you are done).
 
Question: A clock's minute hand has length 4 and its hour hand has length 3. What is the distance between the tips at the moment when it is increasing most rapidly?(Putnam Exam Question) Answer: Making assumption that both the hands moves at constant angular velocities, the answer is ## \sqrt{7} .## But don't you think this assumption is somewhat doubtful and wrong?

Similar threads

  • · Replies 7 ·
Replies
7
Views
4K
  • · Replies 8 ·
Replies
8
Views
2K
Replies
4
Views
2K
  • · Replies 11 ·
Replies
11
Views
3K
  • · Replies 47 ·
2
Replies
47
Views
4K
  • · Replies 11 ·
Replies
11
Views
3K
Replies
2
Views
2K
Replies
5
Views
2K
  • · Replies 4 ·
Replies
4
Views
1K
  • · Replies 1 ·
Replies
1
Views
2K